LSAT and Law School Admissions Forum

Get expert LSAT preparation and law school admissions advice from PowerScore Test Preparation.

 kristinaroz93
  • Posts: 160
  • Joined: Jul 09, 2015
|
#19660
"Politician: All nations that place a high tax on income produce thereby a negative incentive for technological innovation.."

I do not understand how in the conditional chain we equated "nation wants to maintain its value system and way of life" to "not wind up in a strategically disadvantageous position" and "not lose voice in world affairs".

Which elements are equated to what here? I am just not sure how we made those leaps!

At that point if we are making such leaps couldn't we then also say that a "nation (that) wants to maintain its value system and way of life" also could mean "not have negative incentive for technnological innovation", because we can also argue that, that too can affect a nation's "way of life". I just don't understand how it chose those two elements ("not wind up in a strategically disadvantageous position" and "not lose voice in world affairs") to equate to from the chain, just looks like an impossible leap to me. How can I better justify the leap in my head? Is it because those options were both closest to the conclusion? And why did we need both options, could it have just been "not lose voice in world affairs", essenitally how did we pick those two options?

I hope my question is making sense!

Best,

Kristina
 David Boyle
PowerScore Staff
  • PowerScore Staff
  • Posts: 836
  • Joined: Jun 07, 2013
|
#19666
kristinaroz93 wrote:"Politician: All nations that place a high tax on income produce thereby a negative incentive for technological innovation.."

I do not understand how in the conditional chain we equated "nation wants to maintain its value system and way of life" to "not wind up in a strategically disadvantageous position" and "not lose voice in world affairs".

Which elements are equated to what here? I am just not sure how we made those leaps!

At that point if we are making such leaps couldn't we then also say that a "nation (that) wants to maintain its value system and way of life" also could mean "not have negative incentive for technnological innovation", because we can also argue that, that too can affect a nation's "way of life". I just don't understand how it chose those two elements ("not wind up in a strategically disadvantageous position" and "not lose voice in world affairs") to equate to from the chain, just looks like an impossible leap to me. How can I better justify the leap in my head? Is it because those options were both closest to the conclusion? And why did we need both options, could it have just been "not lose voice in world affairs", essenitally how did we pick those two options?

I hope my question is making sense!

Best,

Kristina
Hello,

There may not be a connection between "nation wants to maintain its value system and way of life" and "not wind up in a strategically disadvantageous position" and "not lose voice in world affairs"; that may be the point of answer D.

David
 kristinaroz93
  • Posts: 160
  • Joined: Jul 09, 2015
|
#19668
The LR bible states The phrase "nation wants to maintain its value system and way of life" is a rough equivalent of "not wind up in a strategically disadvantageous position/ not fall behind in arms race and not lose a voice in world affairs".

And then the book does a whole diagrmming portion using that information to prove answer chocie A wrong.

end of stimulus states :"So if a nation wants to maintain its value system and way of life, it must not allow its highest tax bracket to exceed 30 percent of income".

Which was then diagrammed in the bible as:

not fall behind in international arms race and not lose voice in world affairs --> must not allow its highest tax bracket to exceed 30 percent of income.

So there was an equating of terms happening. But it i not clear to me how those terms are equated.
 Nikki Siclunov
PowerScore Staff
  • PowerScore Staff
  • Posts: 1362
  • Joined: Aug 02, 2011
|
#19670
Hi Kristina,

Thanks for your question! These statements are certainly not logically equivalent: the conclusion assumes that they are, but there are clearly multiple gaps in the conditional logic which we can exploit to our advantage. Here's how I would diagram the argument:
Premise 1: High Tax :arrow: Neg Incentive for Tech
Premise 2: Tech Hampered :arrow: Fall behind in Arms Race
Premise 2: Strategic Disadvantage :arrow: Lose Influence

Conclusion: Maintain Way of Life :arrow: Limit Taxes to 30%
The conclusion is merely an attempt to construct a contrapositive from the conditional chain the author assumes can be constructed from the premises. There are several problems with this:

1. The necessary condition of Premise 1 is not identical to the sufficient condition of Premise 2, because simply having a negative incentive for innovation does not mean that such innovation will be necessarily hampered. The author assumes the equivalence of these ideas, an assumption exploited by answer choice (B).

2. The necessary condition of Premise 2 is not identical to the sufficient condition of Premise 3, because falling behind in the arms race is not logically equivalent to having a strategic disadvantage (it's close, but not exactly the same). The author assumes that they are, which is exactly why answer choice (C) weakens the argument.

3. Even if we assume that the premises do form a conditional chain (they do not, but let's assume, for the sake of argument, that they do), the conclusion is still not a precise contrapositive of that chain. The proper conclusion should have been this:

NOT Lose Influence :arrow: NO High Tax

However, this is not the conclusion we have in the stimulus! Not losing our influence in the world isn't quite the same as "maintaining our value system and way of life." The author assumes that these ideas are logically equivalent, an assumption exploited by answer choice (D).

Finally, even if we assume that the two ideas are synonymous, avoiding high taxes is not quite the same as keeping them capped at 30%. Why 30%? Answer choice (A) exploits this weakness, suggesting that as long as income taxes stay below 45%, they would not deter investors.

Each of the four incorrect answer choices in this Weaken-EXCEPT question exploits a different weakness in the argument. Answer choice (E) is irrelevant, and is therefore correct.

Does this clear things up? Let me know.

Thanks! :-)
 kristinaroz93
  • Posts: 160
  • Joined: Jul 09, 2015
|
#19672
Hi Nikki,

This was a very wonderful response, and cleared a lot of things up for me! I now see that it is the author who is forcing the terms to be equated (not the lr bible) and our goal was to break down what he equated so that the chain would break down.

Few more questions:
1) Shouldn't the ACTUAL/CORRECT contrapositive of the chain be: NOT Lose voice in world affairs --->NO High Tax (or is infuence and voice interchangeable here)

1a) So in choice D where it states: lose influence in world community, does it actually mean lose voice in world affairs?

And I actually didn't realize that the last sentence was a contrapositive of the stimulus, just thought it was its own thing, because of how different all the terms were.
2) So the statement the author creates for the contrapositive that he assumes works is:
not lose voice in world affairs-->not allow its highest tax bracket to exceed 30 percent of income, right?

2a) And so when we apply his condtional statement to choice A, the answer is weakened since we can have a situation where we do not lose voice in world affairs, but the highest tax bracket exceeds 30 percent of income. Thus, the sufficient occurs but necc is not met. Is my reasoning correct?

(I just want to see if my understanding of your explanation is correct)

3) And I guess my real confusion was that the lr bible assumed the author equated "nation wants to maintain its value system and way of life" to both "not lose voice in world affairs" and "not fall behind in arms race". I can't seem to figure out how we think the author equated to both those things and not just "not lose voice in world affairs" only.

Best,

Kristina
 Nikki Siclunov
PowerScore Staff
  • PowerScore Staff
  • Posts: 1362
  • Joined: Aug 02, 2011
|
#19674
Hi Kristina,

See my answers below.
1a) So in choice D where it states: lose influence in world community, does it actually mean lose voice in world affairs?
Yes. The two are synonymous.
And I actually didn't realize that the last sentence was a contrapositive of the stimulus, just thought it was its own thing, because of how different all the terms were.
Sometimes, a condition can be restated in other terms, as long as the meaning is close enough. Unfortunately, this particular author takes this a bit too far :)
2) So the statement the author creates for the contrapositive that he assumes works is:
not lose voice in world affairs-->not allow its highest tax bracket to exceed 30 percent of income, right?
Correct.
2a) And so when we apply his conditional statement to choice A, the answer is weakened since we can have a situation where we do not lose voice in world affairs, but the highest tax bracket exceeds 30 percent of income. Thus, the sufficient occurs but necc is not met. Is my reasoning correct?
Sort of. The author argues that high taxes deter investors, and concludes that the tax rate should be limited to 30%. This is an arbitrary limit. By the contrapositive of his premise, it is clear that for investors not to be deterred, taxes should be kept at a reasonable rate (i.e. "not high"). But why at 30%? Answer choice A weakens the argument by showing that even a tax rate that exceeds 30% would not be sufficient to deter investors.
3) And I guess my real confusion was that the lr bible assumed the author equated "nation wants to maintain its value system and way of life" to both "not lose voice in world affairs" and "not fall behind in arms race". I can't seem to figure out how we think the author equated to both those things and not just "not lose voice in world affairs" only.
The LRB does not equate these concepts; if anything, the author does (from the contrapositive of the conditional chain that he thought can be created from his premises).

Good luck!
 kristinaroz93
  • Posts: 160
  • Joined: Jul 09, 2015
|
#19675
Thank you so much I really understand it all better!

(And for the last tibbit I think you misunderstood because I wrote "lr bible assumed the AUTHOR equated "nation wants to maintain its value system and way of life" to both "not lose voice in world affairs" and "not fall behind in arms race". And so I ask why we think the author equated to both those things and not just "not lose voice in world affairs" only?). =)


-Best,

Kristina
 David Boyle
PowerScore Staff
  • PowerScore Staff
  • Posts: 836
  • Joined: Jun 07, 2013
|
#19718
kristinaroz93 wrote:Thank you so much I really understand it all better!

(And for the last tibbit I think you misunderstood because I wrote "lr bible assumed the AUTHOR equated "nation wants to maintain its value system and way of life" to both "not lose voice in world affairs" and "not fall behind in arms race". And so I ask why we think the author equated to both those things and not just "not lose voice in world affairs" only?). =)


-Best,

Kristina
Hello,

Re the last bit, the author may think tax and tech and arms and strategic disadvantage and losing voice and values/way of life are all linked together.

David
 avengingangel
  • Posts: 275
  • Joined: Jun 14, 2016
|
#29587
The Course book says (A) is wrong because: "This attacks the necessary condition of the conclusion by showing that taxes could exceed 30% before problems occurred" But... the conclusion doesn't really say that... it says that it can't exceed 30 percent of income. Over 45% exceeds 30% -- the nec condition is still true.

I chose this answer because, to me, it still held the necessary condition true (of not ALLOWING the tax bracket to exceed 30). I definitely don't want to be that person who wastes time arguing with the test, so... can you provide any additional explanation as to what I should recognize/pay attention to in an answer choice like this, to rule it out? I'm afraid I'm not fully grasping "what not to do" from the Course book's explanation. Thanks!

Also, what PT and question # is this ??
User avatar
 Jonathan Evans
PowerScore Staff
  • PowerScore Staff
  • Posts: 727
  • Joined: Jun 09, 2016
|
#29590
Hi, Avenging,

The conclusion contains just such a necessary condition. As discussed in the LRB, the conclusion is that "if a nation wants to maintain its value system and way of life, then it must not allow its highest bracket to exceed 30%." The "then" clause is the necessary condition of the conclusion.

Answer choice A undermines this necessary condition of the conclusion. When you show that a purported "necessary" condition is not in fact necessary, you undermine the validity of the conclusion.

Briefly, given a conditional A :arrow: B, the conditional is true in the following scenarios:

A true and B true :arrow: true conditional (A implies B)
A not true and B true :arrow: true conditional (A not necessary for B)
A not true and B not true :arrow: true conditional (no problem here either)

However what happens here?

A true and B not true :arrow: false conditional (A implies B, so if A is true and B is false then the conditional is false)

Answer choice A says that 30% need not be the upper limit on tax rate before negative effects occur. Therefore, the 30% ceiling is not in fact necessary. Since it is not necessary, the whole conditional falls apart. The conclusion is invalid.

(PT 35, October 2001, Section 4, Question 17)

Get the most out of your LSAT Prep Plus subscription.

Analyze and track your performance with our Testing and Analytics Package.